2025 AMC 12A Problems/Problem 9: Difference between revisions
Mrenthusiasm (talk | contribs) |
|||
| (5 intermediate revisions by 3 users not shown) | |||
| Line 9: | Line 9: | ||
Values on the complex plane can easily be represented as points on the Cartesian plane, so we go ahead and do that so we are in a more familiar place. | Values on the complex plane can easily be represented as points on the Cartesian plane, so we go ahead and do that so we are in a more familiar place. | ||
Translating onto the Cartesian plane, we have the points <imath>(2,1)</imath> and <imath>(3,4)</imath>. The slope of the line passing through these points is <imath>\frac{4-1}{3-2} = 3</imath>, so the equation of this line is | Translating onto the Cartesian plane, we have the points <imath>(2,1)</imath> and <imath>(3,4)</imath>. The slope of the line passing through these points is <imath>\frac{4-1}{3-2} = 3</imath>, so the equation of this line is | ||
<cmath>y = 3(x-2)+1 | <cmath>\begin{align*} | ||
y &= 3(x-2)+1 \\ | |||
We want the real number that passes through this line, which is equivalent to the <imath>x-</imath>intercept. This occurs when <imath>y=0</imath>, so the <imath>x</imath>-intercept of this line is <imath>x = \boxed{\ | y &= 3x-5. | ||
\end{align*}</cmath> | |||
We want the real number that passes through this line, which is equivalent to the <imath>x-</imath>intercept. This occurs when <imath>y=0</imath>, so the <imath>x</imath>-intercept of this line is <imath>x=\boxed{\textbf{(E)}~\frac53}.</imath> | |||
~lprado (minor edits ~Logibyte) | ~lprado (minor edits ~Logibyte) | ||
| Line 22: | Line 24: | ||
By De Moivre's Theorem, we have <imath>w^2=5\operatorname{cis}(2\theta),</imath> from which <cmath>\tan(2\theta)=\frac{2\tan\theta}{1-\tan^2\theta}=\frac{4}{3}.</cmath> | By De Moivre's Theorem, we have <imath>w^2=5\operatorname{cis}(2\theta),</imath> from which <cmath>\tan(2\theta)=\frac{2\tan\theta}{1-\tan^2\theta}=\frac{4}{3}.</cmath> | ||
We obtain the following diagram: | We obtain the following diagram: | ||
<asy> | |||
/* Made by MRENTHUSIASM */ | |||
size(200); | |||
int xMin = -2; | |||
int xMax = 5; | |||
int yMin = -2; | |||
int yMax = 5; | |||
//Draws the horizontal gridlines | |||
void horizontalLines() | |||
{ | |||
for (int i = yMin+1; i < yMax; ++i) | |||
{ | |||
draw((xMin,i)--(xMax,i), mediumgray+linewidth(0.4)); | |||
} | |||
} | |||
//Draws the vertical gridlines | |||
void verticalLines() | |||
{ | |||
for (int i = xMin+1; i < xMax; ++i) | |||
{ | |||
draw((i,yMin)--(i,yMax), mediumgray+linewidth(0.4)); | |||
} | |||
} | |||
horizontalLines(); | |||
verticalLines(); | |||
draw((xMin,0)--(xMax,0),black+linewidth(1.5),EndArrow(5)); | |||
draw((0,yMin)--(0,yMax),black+linewidth(1.5),EndArrow(5)); | |||
label("Re",(xMax,0),(2,0)); | |||
label("Im",(0,yMax),(0,2)); | |||
pair W1, W2, R; | |||
W1 = (2,1); | |||
W2 = (3,4); | |||
R = (5/3,0); | |||
label("$w=2+i$", W1, 2*dir(W1)); | |||
label("$w^2$", W2, 2*dir(W2)); | |||
label("$r$", R, 2*dir(R-W2)); | |||
label("$\theta$", origin, 6*dir(14)); | |||
label("$\theta$", origin, 6*dir(40)); | |||
draw(W1--origin--W2); | |||
draw(W2--R,dashed); | |||
dot(W1^^W2^^R, linewidth(4)); | |||
</asy> | |||
Since <imath>\left|w^2\right|=5,</imath> we have <imath>w^2=3+4i</imath> by a <imath>3</imath>-<imath>4</imath>-<imath>5</imath> triangle. The complex numbers <imath>w</imath> and <imath>w^2</imath> correspond to the points <imath>(2,1)</imath> and <imath>(3,4),</imath> respectively, and <imath>r</imath> corresponds to the <imath>x</imath>-intercept of this line. In slope-intercept form, the line containing these two points is <imath>y=3x-5.</imath> Therefore, the <imath>x</imath>-intercept is <imath>r=\boxed{\textbf{(E)}~\frac53}.</imath> | Since <imath>\left|w^2\right|=5,</imath> we have <imath>w^2=3+4i</imath> by a <imath>3</imath>-<imath>4</imath>-<imath>5</imath> triangle. The complex numbers <imath>w</imath> and <imath>w^2</imath> correspond to the points <imath>(2,1)</imath> and <imath>(3,4),</imath> respectively, and <imath>r</imath> corresponds to the <imath>x</imath>-intercept of this line. In slope-intercept form, the line containing these two points is <imath>y=3x-5.</imath> Therefore, the <imath>x</imath>-intercept is <imath>r=\boxed{\textbf{(E)}~\frac53}.</imath> | ||
~MRENTHUSIASM | ~MRENTHUSIASM | ||
==Video Solution by Power Solve== | |||
https://www.youtube.com/watch?v=Oy5hEE1fmJU | |||
==Video Solution by SpreadTheMathLove== | |||
https://www.youtube.com/watch?v=dAeyV60Hu5c | |||
==See Also== | ==See Also== | ||
Latest revision as of 11:40, 11 November 2025
Problem
Let
be the complex number
, where
. What real number
has the property that
,
, and
are three collinear points in the complex plane?
Solution 1 (Rectangular Form)
We begin by calculating
:
Values on the complex plane can easily be represented as points on the Cartesian plane, so we go ahead and do that so we are in a more familiar place.
Translating onto the Cartesian plane, we have the points
and
. The slope of the line passing through these points is
, so the equation of this line is
We want the real number that passes through this line, which is equivalent to the
intercept. This occurs when
, so the
-intercept of this line is
~lprado (minor edits ~Logibyte)
Solution 2 (Polar Form)
Recall that the slope of a line is
where
is the angle formed by the line and the positive
-axis.
Note that
In polar coordinates, let
It follows that
By De Moivre's Theorem, we have
from which
We obtain the following diagram:
Since
we have
by a
-
-
triangle. The complex numbers
and
correspond to the points
and
respectively, and
corresponds to the
-intercept of this line. In slope-intercept form, the line containing these two points is
Therefore, the
-intercept is
~MRENTHUSIASM
Video Solution by Power Solve
https://www.youtube.com/watch?v=Oy5hEE1fmJU
Video Solution by SpreadTheMathLove
https://www.youtube.com/watch?v=dAeyV60Hu5c
See Also
| 2025 AMC 12A (Problems • Answer Key • Resources) | |
| Preceded by Problem 8 |
Followed by Problem 10 |
| 1 • 2 • 3 • 4 • 5 • 6 • 7 • 8 • 9 • 10 • 11 • 12 • 13 • 14 • 15 • 16 • 17 • 18 • 19 • 20 • 21 • 22 • 23 • 24 • 25 | |
| All AMC 12 Problems and Solutions | |
These problems are copyrighted © by the Mathematical Association of America.